Thursday, June 12, 2008

Mistake in question 2

In home assignment #5, there is an error in question 2. Instead of the probability that $|f(x)|>t$, the bound should be on the probability that $|f(x)|>t\cdot||f||_2$. Thanks to Ori Brostovski for pointing that out.

No comments: